Autor Tema: Historia de una prueba sencilla del UTF y petición de ayuda para su revisión.

0 Usuarios y 1 Visitante están viendo este tema.

12 Septiembre, 2012, 11:55 am
Respuesta #80

aureodd

  • $$\Large \color{#5e8d56}\pi\,\pi\,\pi$$
  • Mensajes: 241
  • País: es
  • Karma: +0/-0
  • Sexo: Masculino
    • Matemáticas Recreativas, Juegos, Lógica, Ingenio, Matemáticos
Hola el_manco,
entonces si
\( 3^{2m-3}(3^2p^3-(2p)^3)=3^{2m-3}p^3 \) es múltiplo de \( u \)
\( u=3^{2m-3} \) ya que \( q,r  \) no son múltiplos de \( 3 \) y además \( u \) no puede dividir a \( p \)
Prueba:
Si \( u \) divide a \( p \) podemos poner \( p=ut \) para algún \( t \). Sutituimos en
\( qru=3^{2m-3}(3^2p^3-(2p+u)^3) \)
nos queda que el término de la derecha es múltiplo de \( u^3 \) y no es posible ya que \( q,r,p \) son coprimos (u no puede dividir a \( q,r \)).
Luego \( u=3^{2m-3} \) y \(  k=2p+3^{2m-3} \) (1)
¿Es correcto?

Por otro lado teniamos
\( qr(k-2p)=3^{2m-3}(3^2p^3-k^3) \) (2)
con
\(  k=2p+u \)
(Si \( k<2p \) el lado izq. en (2) sería negativo y el derecho positivo.
También \( k<3p \) ya que si \( k>3p \) el lado izq. en (2) sería positivo y el derecho negativo.)
Sutituyendo ahora el valor de \( k \) de (1) en (2)
\( qr\cdot3^{2m-3}=3^{2m-3}(3^2p^3-(2p+3^{2m-3})^3)\Rightarrow \)
\( qr=3^2p^3-(2p+3^{2m-3})^3 \) (3)
Como \(  k=2p+3^{2m-3} <3p \Rightarrow 3^{2m-3}<p \)
Y esto hace que el lado derecho de (3) sea negativo, que no es posible ya que \( q.r \) es positivo.
Prueba:
Supongamos \( 3^{2m-3}=p \) y lo sustituimos en (3)
\( qr=3^2(3^{2m-3})^3-(2(3^{2m-3})+3^{2m-3})^3 \Rightarrow \)
\( qr=3^2(3^{2m-3})^3-((2+1).(3^{2m-3})^3 \Rightarrow \)
\( qr=(3^{2m-3})^3(3^2-3^3) \)
¿Es correcto?

Como siempre, muchas gracias por tus respuestas y comentarios.
Saludos

12 Septiembre, 2012, 01:29 pm
Respuesta #81

Luis Fuentes

  • el_manco
  • Administrador
  • Mensajes: 55,996
  • País: es
  • Karma: +0/-0
Hola

entonces si
\( 3^{2m-3}(3^2p^3-(2p)^3)=3^{2m-3}p^3 \) es múltiplo de \( u \)
\( u=3^{2m-3} \) ya que \( q,r  \) no son múltiplos de \( 3 \) y además \( u \) no puede dividir a \( p \)
Prueba:
Si \( u \) divide a \( p \) podemos poner \( p=ut \) para algún \( t \). Sutituimos en
\( qru=3^{2m-3}(3^2p^3-(2p+u)^3) \)
nos queda que el término de la derecha es múltiplo de \( u^3 \) y no es posible ya que \( q,r,p \) son coprimos (u no puede dividir a \( q,r \)).
Luego \( u=3^{2m-3} \) y \(  k=2p+3^{2m-3} \) (1)
¿Es correcto?

Está bien. Aunque el razonamiento no es del todo correcto. Que \( qru \) sea múltiplo de \( u^3 \) no quiere decir ni que \( q \) ni que \( r \) sean necesariamente múltiplos de \( u \); si que tengan factores primos comunes. Eso esta imposiblitado por la coprimalidad de \( q,r,p \) y el supuesto de \( p=ut \).

Además lo que razonas así es que \( p \) no es múltipo de \( u \); pero eso no excluye que \( u,p \) pudieran tener factores primos comunes. Es decir de lo que has hecho no se deduce necesariamante que \( u=3^{2m-3} \). Lo que debes de hacer es lo mismo que has hecho con \( u \), pero con un factor primo común de mayor exponente posible  del \( m.c.d.(p,u) \).


Citar
Por otro lado teniamos
\( qr(k-2p)=3^{2m-3}(3^2p^3-k^3) \) (2)
con
\(  k=2p+u \)
(Si \( k<2p \) el lado izq. en (2) sería negativo y el derecho positivo.
También \( k<3p \) ya que si \( k>3p \) el lado izq. en (2) sería positivo y el derecho negativo.)
Sutituyendo ahora el valor de \( k \) de (1) en (2)
\( qr\cdot3^{2m-3}=3^{2m-3}(3^2p^3-(2p+3^{2m-3})^3)\Rightarrow \)
\( qr=3^2p^3-(2p+3^{2m-3})^3 \) (3)
Como \(  k=2p+3^{2m-3} <3p \Rightarrow 3^{2m-3}<p \)
Y esto hace que el lado derecho de (3) sea negativo, que no es posible ya que \( q.r \) es positivo.
Prueba:
Supongamos \( 3^{2m-3}=p \) y lo sustituimos en (3)
\( qr=3^2(3^{2m-3})^3-(2(3^{2m-3})+3^{2m-3})^3 \Rightarrow \)
\( qr=3^2(3^{2m-3})^3-((2+1).(3^{2m-3})^3 \Rightarrow \)
\( qr=(3^{2m-3})^3(3^2-3^3) \)
¿Es correcto?

El problema de todo esto es que no tengo claro que \( p,q,r \) tengan que ser positivos. Hay que echar la vista atrás y ver de donde viene la expresión con la que trabajas. En el razonamiento que yo había sintetizado de los tuyos:


Spoiler
Citar
En primer lugar es cómodo escribir la ecuación de Fermat como \( x^3+y^3+z^3=0 \) en lugar de la manera clásica, permitiendo que las variables sean negativas. Esto evita tener que repetir argumentos para tus tres términos \( (x-y),(x-z)=(y-b),M=(y+z) \) que ahora con mi notación son simplemente \( (x+y),(x+z),(y+z) \). De forma que por simetría lo que probemos para uno, queda probado para los demás.

 Entonces sean \( x,y,z \) tres enteros coprimos dos a dos verificando \( x^3+y^3+z^3=0. \)

 1) \( 3(x+y)(x+z)(y+z)=(x+y+z)^3 \)

 Prueba: Basta tener en cuenta que, \( 3(x+y)(x+z)(y+z)=(x+y+z)^3-x^3-y^3-z^3 \) y usar que estamos bajo el supuesto de que los tres números cumplen la ecuación de Fermat.

 2) \( (x+y),(x+z),(y+z) \) son coprimos dos a dos.

 Prueba: Basta tener en cuenta que por (1) cualquier divisor primo de uno de esos términos lo es de \( (x+y+z) \). Entonces si \( x+y,x+z \) son divisibles por \( p \),

 \( z=(x+y+z)-(x+y) \) divisible por \( p \)
 \( y=(x+y+z)-(x+z) \) divisible por \( p \)

 Pero \( z,y \) son coprimos: contradicción.

 3) Uno y sólo uno de los términos \( (x+y),(x+z),(y+z) \) es divisible por \( 3 \) (supondremos sin pérdida de generalidad, a partir de ahora, que tal término es \( y+z \)).

 Prueba: Por (1), (\( x+y+z) \) es divisible por \( 3 \) y por tanto \( 3(x+y)(x+z)(y+z)=(x+y+z)^3 \) divisible por \( 3^3 \). Por tanto efecivamente alguno de los términos es divisible por tres. La unicidad es consecuencia de (2).

 4) \( (y+z)=3^mE^3 \) con \( mcd(E,3)=1 \) y \( m\equiv 2 \) mod \( 3 \).

 Prueba: Se tiene que \(  -x^3=y^3+z^3=(y+z)(y^2-yz+z^2)  \). Como \( (y+z) \) es múltiplo de tres, también lo es \( x \). Como \( x,y,z \) son coprimos, \( y,z \) no son divsibles por tres. Por tanto:

\( (y+z)=3^mE^3,\qquad (y^2-yz+z^2)=3^{3k-m}F^3 \) con \( mcd(E,3)=mcd(F,3)=1 \).

 Pero: \( 3yz=(y+z)^2-(y^2-yz+z^2)=3^{2m}E^6-3^{3k-m}F^3 \). Como \( y,z \) no son divisibles por tres necesariamente \( 3k-m=1 \), es decir, \( m\equiv 2 \) mod \( 3 \).

 5) Existen enteros \( p,q,r  \) coprimos dos a dos y no múltiplos de \( 3 \) tales que:
 
\( y+z=3^mp^3 \) con \( m\equiv 2 \) mod \( 3 \)
\( x+y=q^3\quad x+z=r^3\quad x+y+z=3^{\frac{m+1}{3}}pqr \)

 Prueba: Basta aplicar (2),(3),(4) a (1)

 6) Existen enteros \( p,q,r  \) coprimos dos a dos y no múltiplos de \( 3 \) tales que:

\(  q^3=2\cdot 3^{\frac{m+1}{3}}pqr-3^mp^3-r^3 \) con \( m\equiv 2 \) mod \( 3 \)

 Prueba: Basta tener en cuenta que por (5):

\(  x=(x+y+z)-(y+z)=3^{\frac{m+1}{3}}pqr-3^mp^3 \)
\(  y=(x+y+z)-(x+z)=3^{\frac{m+1}{3}}pqr-r^3 \)
\(  x+y=q^3 \)
[cerrar]

 \( p,q,r \), eran enteros pero no necesariamente positivos.

Saludos.

14 Septiembre, 2012, 01:05 am
Respuesta #82

aureodd

  • $$\Large \color{#5e8d56}\pi\,\pi\,\pi$$
  • Mensajes: 241
  • País: es
  • Karma: +0/-0
  • Sexo: Masculino
    • Matemáticas Recreativas, Juegos, Lógica, Ingenio, Matemáticos
Hola,
Citar
El problema de todo esto es que no tengo claro que \( p,q,r \) tengan que ser positivos. Hay que echar la vista atrás y ver de donde viene la expresión con la que trabajas.
Todo viene de la notación clásica del UTF
\( x^n=y^n+z^n \)
utilizando que \( x,y,z \) son enteros positivos
Con esto había llegado a los siguientes resultados:

I) \( (3^{3m-1}p^3+r^3+3^{m}pqr)^3=(3^{3m-1}p^3+3^{m}pqr)^3+(r^3+3^{m}pqr)^3 \)
donde
\( x^3=(3^{3m-1}p^3+r^3+3^{m}pqr)^3 \)
\( y^3=(3^{3m-1}p^3+3^{m}pqr)^3 \)
\( z^3=(r^3+3^{m}pqr)^3 \)

II) \( (q^3-3^{m}pqr)^3=(3^{3m-1}p^3+3^{m}pqr)^3+(r^3+3^{m}pqr)^3 \)

Utilizando I) en II)
\( (3^{3m-1}p^3+r^3+3^{m}pqr)^3=(q^3-3^{m}pqr)^n \)
obtengo el resultado en III)

III) \( q^3= 3^{3m-1}p^3+r^3+2\cdot3^{m}pqr \)
Y este último es el que utilizado en los desarrollos de los post anteriores con \( p,q,r \) positivos.

En el documento para \( n=3 \) en el enlace http://eduardoochoa.com/joomla/content/view/569/68/1/1/
se puede seguir la línea del desarrollo pero modificando según comentaste:
 

 Allí el término \( 3pqr \) pasaría a \( 3^{\frac{2m+1}{3}} \) y el término \( 3^2p^3 \) pasaría a \( 3^{2m}p^3 \).

 Esto hace desaparecer la contradicción con la que concluye la prueba del Lema 6.

Es equivalente a III) haciendo el cambio de variable \( \frac{2m+1}{3}=j \Rightarrow 3j-1=2m \)

Hola

entonces si
\( 3^{2m-3}(3^2p^3-(2p)^3)=3^{2m-3}p^3 \) es múltiplo de \( u \)
\( u=3^{2m-3} \) ya que \( q,r  \) no son múltiplos de \( 3 \) y además \( u \) no puede dividir a \( p \)
Prueba:
Si \( u \) divide a \( p \) podemos poner \( p=ut \) para algún \( t \). Sutituimos en
\( qru=3^{2m-3}(3^2p^3-(2p+u)^3) \)
nos queda que el término de la derecha es múltiplo de \( u^3 \) y no es posible ya que \( q,r,p \) son coprimos (u no puede dividir a \( q,r \)).
Luego \( u=3^{2m-3} \) y \(  k=2p+3^{2m-3} \) (1)
¿Es correcto?

Está bien. Aunque el razonamiento no es del todo correcto. Que \( qru \) sea múltiplo de \( u^3 \) no quiere decir ni que \( q \) ni que \( r \) sean necesariamente múltiplos de \( u \); si que tengan factores primos comunes. Eso esta imposiblitado por la coprimalidad de \( q,r,p \) y el supuesto de \( p=ut \).

Además lo que razonas así es que \( p \) no es múltipo de \( u \); pero eso no excluye que \( u,p \) pudieran tener factores primos comunes. Es decir de lo que has hecho no se deduce necesariamante que \( u=3^{2m-3} \). Lo que debes de hacer es lo mismo que has hecho con \( u \), pero con un factor primo común de mayor exponente posible  del \( m.c.d.(p,u) \).


Si \( u,p \) tienen factores comunes, para que sea posible que \( u|3^{2m-3}p^3 \) se tiene que dar que \( u>p \)?
Por ejemplo para \( m=3 \) tomo:
\( u=3^3\cdot5^3 \)
\( p=5 \)
Y se cumple también lo que teníamos en
\( qr=3^2p^3-(2p+u)^3 \) (3)
Y esto hace que el lado derecho de (3) sea negativo, que no es posible ya que \( q.r \) es positivo.
Es correcto?
Muchas gracias!!!!
Saludos

14 Septiembre, 2012, 11:00 am
Respuesta #83

Luis Fuentes

  • el_manco
  • Administrador
  • Mensajes: 55,996
  • País: es
  • Karma: +0/-0
Hola

Si \( u,p \) tienen factores comunes, para que sea posible que \( u|3^{2m-3}p^3 \) se tiene que dar que \( u>p \)?
Por ejemplo para \( m=3 \) tomo:
\( u=3^3\cdot5^3 \)
\( p=5 \)

No entiendo muy bien que quieres decir con esto. En principio para que \( u|3^{2m-3}p^3 \) no veo porque \( u>p \). En el mismo ejemplo que pones tu \( p \) podría ser \( 5\cdot 1231243212345145145 \).

De todas formas aunque maticé la prueba que hiciste, es correcto concluir que \( u=2^{2m-3} \)-

Citar
\( qr=3^2p^3-(2p+u)^3 \) (3)
Y esto hace que el lado derecho de (3) sea negativo, que no es posible ya que \( q.r \) es positivo.
Es correcto?
[/quote]

No estoy seguro de que me estás preguntando aquí:

- Si \( u>p \) entonces es cierto que el lado derecho de la igualdad es negativo (pero como dije antes no veo como probar que \( u>p \)).

- Si como en tu anterior intervención suponemos \( u=2^{2m-3} \) y suponemos \( p>3^{2m-3}=u \), no es cierto que el lado derecho de la igualdad tenga que ser negativo.

Saludos.

25 Septiembre, 2012, 11:13 am
Respuesta #84

aureodd

  • $$\Large \color{#5e8d56}\pi\,\pi\,\pi$$
  • Mensajes: 241
  • País: es
  • Karma: +0/-0
  • Sexo: Masculino
    • Matemáticas Recreativas, Juegos, Lógica, Ingenio, Matemáticos
Hola el_manco,
es correcto lo que comentas:
Si \( p>3^{2m-3}=u \), no es cierto que el lado derecho de la igualdad tenga que ser negativo.
y de momento no creo que sirva de mucho...

Teníamos entonces
\( q=3^{m-1}k+r \) para algún \( k \)
y que
\( u=3^{2m-3} \)
\(  k=2p+3^{2m-3} \)
Luego \( q-r=3^{m-1}(2p+3^{2m-3}) \) (1)
También teníamos
\( qr=3^2p^3-(2p+3^{2m-3})^3 \) (2)
Luego tenemos el siguiente sistema
\(  \left\{ \begin{array}{c}q-r= 3^{m-1}(2p+3^{2m-3}) \\ qr=3^2p^3-(2p+3^{2m-3})^3 \end{array}\right  \)
De (1)
\( q=3^{m-1}(2p+3^{2m-3})+r \)
Lo sustituimos en (2)
\( (3^{m-1}(2p+3^{2m-3})+r)r=3^2p^3-(2p+3^{2m-3})^3 \)
\( r^2+3^{m-1}(2p+3^{2m-3})r-(3^2p^3-(2p+3^{2m-3})^3)=0 \)
Si llamo \( A=2p+3^{2m-3} \)
\( r^2+3^{m-1}Ar-(3^2p^3-A^3)=0 \)
Podemos buscar soluciones para la ecuación anterior de segundo grado para \( r \):
\( r = \dfrac{{ - b \pm \sqrt {b^2 - 4ac} }}{2a} \)

\( r = \dfrac{{3^{m-1}A \pm \sqrt {(3^{m-1}A)^2 +4(3^2p^3-A^3)} }}{2} \)

donde \( (3^{m-1}A)^2 +4(3^2p^3-A^3) \)
tiene que ser un cuadrado.
¿Es correcto hasta aquí?
Muchas gracias!!!!
Saludos

25 Septiembre, 2012, 12:51 pm
Respuesta #85

Luis Fuentes

  • el_manco
  • Administrador
  • Mensajes: 55,996
  • País: es
  • Karma: +0/-0

26 Septiembre, 2012, 10:46 am
Respuesta #86

aureodd

  • $$\Large \color{#5e8d56}\pi\,\pi\,\pi$$
  • Mensajes: 241
  • País: es
  • Karma: +0/-0
  • Sexo: Masculino
    • Matemáticas Recreativas, Juegos, Lógica, Ingenio, Matemáticos
Hola el_manco,
entonces si
\( (3^{m-1}A)^2 +4(3^2p^3-A^3) \)
tiene que ser un cuadrado, con \( A=2p+3^{2m-3} \), y esto es equivalente (si no me he equivocado con las cuentas...) a
\( 4\cdot3^2p^3=4A^3+k^3\cdot3^{m-1}A \) para algún \( k \)
¿Se puede dar que \( A=2p+3^{2m-3} \) divida a \( 4\cdot3^2p^3 \), con \( p \) no múltiplo de \( 3 \)?
Muchas gracias!!!!
Saludos

26 Septiembre, 2012, 12:06 pm
Respuesta #87

aureodd

  • $$\Large \color{#5e8d56}\pi\,\pi\,\pi$$
  • Mensajes: 241
  • País: es
  • Karma: +0/-0
  • Sexo: Masculino
    • Matemáticas Recreativas, Juegos, Lógica, Ingenio, Matemáticos
Hola,
las cuentas que faltaban en la respuesta anterior:
Como \( (3^{m-1}A)^2 +4(3^2p^3-A^3) \)
es un cuadrado mayor que \( (3^{m-1}A)^2 \), lo llamo \( s \),
lo podemos poner como \( (3^{m-1}A)^2+k\cdot3^{m-1}A.k^2 \) para algún \( k \)
es decir,
\( (3^{m-1}A)^2+k^3\cdot3^{m-1}A=s^2 \)
luego
\( k^3\cdot3^{m-1}A=4(3^2p^3-A^3) \)
que es el resultado de la respuesta anterior
..y esto es equivalente (si no me he equivocado con las cuentas...) a
\( 4\cdot3^2p^3=4A^3+k^3\cdot3^{m-1}A \) para algún \( k \)
y sobre el que había planteado la cuestión

¿Se puede dar que \( A=2p+3^{2m-3} \) divida a \( 4\cdot3^2p^3 \), con \( p \) no múltiplo de \( 3 \)?

Muchas gracias de nuevo!!!!
Saludos

26 Septiembre, 2012, 12:58 pm
Respuesta #88

Luis Fuentes

  • el_manco
  • Administrador
  • Mensajes: 55,996
  • País: es
  • Karma: +0/-0
Hola

 No entiendo bien las cuentas que haces aquí:

Hola,
las cuentas que faltaban en la respuesta anterior:
Como \( (3^{m-1}A)^2 +4(3^2p^3-A^3) \)
es un cuadrado mayor que \( (3^{m-1}A)^2 \), lo llamo \( s \),

¿A quién llamas \( s \)? ¿ \( s=3^{m-1}A^2+4(3^2p^3-A^3) \) ?

Citar
lo podemos poner como \( (3^{m-1}A)^2+k\cdot3^{m-1}A.k^2 \) para algún \( k \)
es decir,
\( (3^{m-1}A)^2+k^3\cdot3^{m-1}A=s^2 \)

Y aquí me perdí por completo. ¿De dónde sale esa \( k \)?.

En cualquier caso y respecto a esto:

¿Se puede dar que \( A=2p+3^{2m-3} \) divida a \( 4\cdot3^2p^3 \), con \( p \) no múltiplo de \( 3 \)?

¿Por qué no podría darse?. Podria ocurrir:

\( (2p+3^{2m-3})w=4\cdot 3^2p^3 \)

con \( w \) múltilpo de \( 3^2 \).

Saludos.

26 Septiembre, 2012, 02:08 pm
Respuesta #89

aureodd

  • $$\Large \color{#5e8d56}\pi\,\pi\,\pi$$
  • Mensajes: 241
  • País: es
  • Karma: +0/-0
  • Sexo: Masculino
    • Matemáticas Recreativas, Juegos, Lógica, Ingenio, Matemáticos
Hola el_manco,
sí, yo también me he perdido con las cuentas :banghead: mil perdones!!!
Todas mal!
He llamado \( s^2=(3^{m-1}A)^2 +4(3^2p^3-A^3) \) al cuadrado mayor que \( (3^{m-1}A)^2 \)
y lo tenía que haber escrito como
\( s^2=(3^{m-1}A+k)^2=(3^{m-1}A)^2+2\cdot3^{m-1}A.k+k^2 \)
luego
\( 2\cdot3^{m-1}A.k+k^2=4(3^2p^3-A^3) \)
y no se llega a ningún sitio :(
Disculpa otra vez por la falta de rigurosidad... La próxima vez leeré de alguna lista los primeros 1000 primos y lo volveré a revisar con mas detalle antes de publicarlo ;)
Saludos